Orthogonal trajectories in polar coordinates

Click For Summary
The discussion focuses on finding the orthogonal trajectories to the polar curve described by the equation r² = a²cos(θ). The initial steps involve differentiating the equation with respect to θ and simplifying to obtain dr/dθ = -1/2 tan(θ). A point of confusion arises regarding a step in the book that suggests setting dr/dθ = -r²(dθ/dr), which seems unclear. Clarification reveals that this notation is an abuse, as it simplifies the relationship between the two curves at their intersection points. Ultimately, the explanation resolves the confusion about the orthogonal trajectories in polar coordinates.
patric44
Messages
308
Reaction score
40
Homework Statement
in the polar coordinates find the set of curves that intersects with a right angle with the set of curves describes by r^{2} = a^{2}cos(theta)
Relevant Equations
r^{2} = a^{2}cos(θ)
there is a problem in a book that asks to find the orthogonal trajectories to the curves described by the equation :
$$r^{2} = a^{2}\cos(\theta)$$
the attempt of a solution is as following :
1- i defferntiate with respect to ##\theta## :
$$2r \frac{dr}{d\theta} = -a^{2}\;\sin(\theta)$$
2- i eliminated "a" from the two equations and get :
$$\frac{dr}{d\theta} = -\frac{1}{2}tan(\theta)$$
then the book said to set ##\frac{dr}{d\theta} = -r^{2}\frac{d\theta}{dr} ## ! , this step i don't get ? why would i do that , it doesn't seem to be equal ?!
 
Physics news on Phys.org
patric44 said:
Homework Statement:: in the polar coordinates find the set of curves that intersects with a right angle with the set of curves describes by r^{2} = a^{2}cos(theta)
Relevant Equations:: r^{2} = a^{2}cos(θ)

there is a problem in a book that asks to find the orthogonal trajectories to the curves described by the equation :
$$r^{2} = a^{2}\cos(\theta)$$
the attempt of a solution is as following :
1- i defferntiate with respect to ##\theta## :
$$2r \frac{dr}{d\theta} = -a^{2}\;\sin(\theta)$$
2- i eliminated "a" from the two equations

<br /> \frac{dr}{d\theta} = -\frac12 \tan\theta

I wouldn't descrive that as "eliminating a", so much as "eliminating r".

then the book said to set ##\frac{dr}{d\theta} = -r^{2}\frac{d\theta}{dr} ## ! , this step i don't get ? why would i do that , it doesn't seem to be equal ?!

There is an abuse of notation here.

A curve r = f(\theta) in plane polar coordinates is given by \mathbf{r}(\theta) = f(\theta)\mathbf{e}_r(\theta). The tangent vector is, by the product rule, given by <br /> \frac{d\mathbf{r}}{d\theta} = \frac{df}{d\theta}\mathbf{e}_r + f(\theta) \mathbf{e}_\theta. If another curve r = g(\theta) is orthogonal to this, then we must have <br /> \frac{df}{d\theta} \frac{dg}{d\theta} + fg = 0, but because the curves intersect we have f(\theta) = g(\theta) at this point, and so <br /> \frac{df}{d\theta} = - g(\theta)^2 \frac{d\theta}{dg}. By an abuse of notation which suppress the fact thatr is given by a different function of \theta on each side, this could be written <br /> \frac{dr}{d\theta} = -r^2 \frac{d\theta}{dr}.
 
Last edited:
  • Like
Likes patric44 and etotheipi
thank you so much, it make sense now:smile:
 
Question: A clock's minute hand has length 4 and its hour hand has length 3. What is the distance between the tips at the moment when it is increasing most rapidly?(Putnam Exam Question) Answer: Making assumption that both the hands moves at constant angular velocities, the answer is ## \sqrt{7} .## But don't you think this assumption is somewhat doubtful and wrong?

Similar threads

  • · Replies 19 ·
Replies
19
Views
2K
  • · Replies 14 ·
Replies
14
Views
2K
  • · Replies 5 ·
Replies
5
Views
2K
  • · Replies 1 ·
Replies
1
Views
2K
Replies
3
Views
2K
  • · Replies 4 ·
Replies
4
Views
1K
  • · Replies 14 ·
Replies
14
Views
2K
  • · Replies 5 ·
Replies
5
Views
2K
  • · Replies 3 ·
Replies
3
Views
2K
  • · Replies 8 ·
Replies
8
Views
2K